sportsfan8491
Thanks Received: 12
Forum Guests
 
Posts: 22
Joined: August 28th, 2013
 
 
 

Q6 - The Transit Authority's proposal

by sportsfan8491 Thu Oct 10, 2013 1:21 am

I'd like to take a stab at providing a succinct answer to this one.

The argument seems to be pretty simple to comprehend. The conclusion consists of a prescriptive statement about what must be done: implement a 40% fare increase.

In order to support her conclusion, the author concedes a point about the 40% increase causing some turmoil for the current ridership in the form of an increased cost for taking the bus or subway ("cost" was derived from a common sense perspective because what else would it be). However, the author then goes on to say that not increasing the fares, which she clearly views as the second possible alternative, would not be feasible because the outcome of this "not doing something" alternative would presumably be worse than the 40% increase alternative.

One quick thought that came to mind, before I went to the answer choices, was that the author made several leaps in her argument, basically in going from her premises to her conclusion. It appears she has taken the existence of only two options to be her only consideration in addressing the problem at hand (a bit of a false dichotomy, if you ask me). However, does she conclusively prove that the increase "must" be 40% or are we ever told these these are the "only" two alternatives? If 40% is going to cause some grief, then why can't it simply be a 5% or 10% increase? Based on the evidence presented, I don't see how she definitively proves to us that 40% is the ideal number.

Also, couldn't the Transit Authority cut overhead expenses (i.e. in the form of excessively "fat" executive salaries) in order to find the additional money they need? Ironically, the author doesn't consider this potential third alternative, as is often the case in the real world. Based on these considerations, I wouldn't say the argument is a valid one.

The above discussion isn't required for answering this method of reasoning question, but you never know if you'll see this type of reasoning structure come up in a flaw or assumption question, so I find that a quick argument analysis can be pretty beneficial. And you never know, it might help you rule out an answer choice or two. Now, on to the answers:

A) is wrong because the author doesn't actually employ this argumentative strategy as she concedes that it "will impose some hardship". If you read the stimulus carefully, you can see how blatantly wrong this answer choice is.

B) is wrong because the author doesn't show that the proponent of "any" alternative would be led into a contradiction. She only discusses two alternatives, not every single alternative out there, so if I came in and said: "lets cut some of the excess "fat" in terms of executive salaries!", I wouldn't be led into a contradiction because the author wouldn't have even addressed my alternative. This is an example of where my upfront work in analyzing the argument before going to the answer choices allowed me to quickly see the glaring issue with this answer choice (i.e. "ANY" alternative) and to rule it out almost instantaneously.

C) is perfect. The author rules out the "not increasing the fare" alternative by providing the strongly worded negative consequences associated with it, in an attempt to support her conclusion indirectly. Notice how she doesn't present any direct evidence for her prescriptive 40% fare increase conclusion, so this is why "indirectly" makes sense here.

D) is wrong because the author doesn't even present this comparison in her argument. She leaves the two courses of action or alternatives relatively apart from one another when considering their disadvantages (some financial hardship on riders vs. "severe" cuts to service and losing an "unacceptably" large amount of riders). Thus, it's impossible for us to say that one would not be subject to the same criticism as the other because no overlap between the disadvantages is even discussed in the passage to begin with.

E) is wrong because no form of temporal consistency is discussed or presented in the argument, so this answer can be eliminated very quickly.

I hope my explanation is helpful.
Last edited by sportsfan8491 on Sat Dec 14, 2013 12:06 pm, edited 1 time in total.
User avatar
 
ohthatpatrick
Thanks Received: 3808
Atticus Finch
Atticus Finch
 
Posts: 4661
Joined: April 01st, 2011
 
 
 

Re: Q6 - The Transit Authority's proposal

by ohthatpatrick Fri Oct 11, 2013 2:40 am

Great explanation!

For anyone else reading this, if any part of that didn't click, feel free to ask follow-ups.
 
90kimsj
Thanks Received: 0
Vinny Gambini
Vinny Gambini
 
Posts: 3
Joined: September 25th, 2014
 
 
 

Re: Q6 - The Transit Authority's proposal

by 90kimsj Sun Sep 28, 2014 5:06 am

Thanks for a great explanation.
I have a question though.

For answer choice (D), In interpreted it as the recommended course of action being "increase fares by 40%" and the alternative being "not increasing the fare". So from the argument it can be inferred that not increasing fares can do worse harm than increasing fares, preempting the recommended course of action from being subject to an objection saying "increase fares by 40% would impose a hardship on some riders". So I thought answer choice (D) makes sense.

However, as I was reading this answer choice, I didn't quite fully comprehend the meaning of the answer choice. So what I described above may make no sense. So I would appreciate if some one can explain what the answer choice (D) really means and how it is an incorrect answer.

Thank you.